45-я Международная Математическая Oлимпиада
Греция, Афины, 2004 год


Пусть $n\ge 3$ натуральное число. Пусть ${{t}_{1}}$, ${{t}_{2}}$, $\ldots $, ${{t}_{n}}$ — положительные действительные числа такие, что $${{n}^{2}}+1 > \left( {{t}_{1}}+{{t}_{2}}+\dots+{{t}_{n}} \right)\left( \dfrac{1}{{{t}_{1}}}+\dfrac{1}{{{t}_{2}}}+\dots+\dfrac{1}{{{t}_{n}}} \right).$$ Докажите, что числа ${{t}_{i}}$, ${{t}_{j}}$, ${{t}_{k}}$ являются длинами сторон треугольника при всех $i$, $j$, $k$ таких, что $1\le i < j < k\le n$.
посмотреть в олимпиаде

Комментарий/решение:

пред. Правка 2   0
2023-12-05 20:44:12.0 #

Лемма: если имеются числа $x,y,z>0$ и для них выполняется условие $x+y+z+2=xyz, \ 6<x+y+z<7$ где $n \geq 3$ натуральное число .

Покажем что тогда $x>1,y>1,z>1$

Доказательство: очевидно что $x,y,z<1$ быть не может иначе $x+y+z<3$ рассмотрим случаи

1) Выразим $x=\dfrac{y+z+2}{yz-1}$ и подставим в двойное неравенство

Первый случай:

$x>1, \ z>1, \ 0<y<1$

1) $ y+z+ \dfrac{y+z+2}{yz-1} > 6 $ или $\dfrac{yz(y+z-6)+8}{yz-1} > 0$ так как $z>1,0<y<1$ или $(z-1)(1-y)>0$ откуда $y+z>yz+1$ значит $yz(y+z-6)+8>yz(yz-5)+8 = (yz-\dfrac{5}{2})^2 + \dfrac{7}{4} > 0$ откуда $yz>1$

2) $y+z+\dfrac{y+z+2}{yz-1} < 7$ или $\dfrac{yz(y+z-7)+9}{yz-1} < 0$ но учитывая $y+z>yz+1$ тогда $yz(y+z-7)+9>yz(yz-6)+9 = (yz-3)^2>0$ где $yz \ne 3$ иначе $y>1$ тогда $yz<1$ противоречие .

Второй случай:

$x>1,0<z<1,0<y<1$

1) $y+z+\dfrac{y+z+2}{yz-1} > 6$ или $\dfrac{yz(y+z-6)+8}{yz-1} > 0 $ и так как $(1-z)(1-y)>0$ или $1+yz>y+z$ тогда $yz(y+z-6)+8 > (y+z-1)(y+z-6)+8 = (y+z-\dfrac{7}{2})^2+\dfrac{7}{4} > 0$ но $yz<1$ откуда $\dfrac{yz(y+z-6)+8}{yz-1} < 0$

значит $x>1, y>1 , z>1$

К задаче:

По КБШ получаем что ${{n}^{2}}+1 > \left( {{t}_{1}}+{{t}_{2}}+\dots+{{t}_{n}} \right)\left( \dfrac{1}{{{t}_{1}}}+\dfrac{1}{{{t}_{2}}}+\dots+\dfrac{1}{{{t}_{n}}} \right) \geq n^2$ тогда открыв скобки, собрав и перенеся только числовые значения, если $S$ сумма буквенных выражений(дробей) получается $n^2-n \leq S < n^2-n+1$ $(*)$ отметим что среди $S$ всегда можно выбрать условные $t_{i}=x, t_{j}=y, t_{k}=z$ для любых $1\le i < j < k\le n$ такие что $S_{1}=\dfrac{x}{y}+\dfrac{z}{y} , \ S_{2}=\dfrac{y}{x}+\dfrac{z}{x}, \ S_{3}=\dfrac{x}{z}+\dfrac{y}{z}$

Тогда отметим что по $AM \geq GM$ получается $6<A_{1}=S_{1}+S_{2}+S_{3}$ так же $A_{1}<7$ иначе для остальных дробей $A_{2}$ в $S$ учитывая $(*)$ выполняется

$n^2-n-6<A_{2}<n^2-n+1-v$ где $v>7$ что неверно.

Тогда применяя лемму для $A_{1}$ для которой выполняется все требования, только в данном случае $S_{1}=x, \ S_{2}=y, \ S_{3}=z$ получается что $S_{1}, \ S_{2}, \ S_{3}>1$ тогда $x+z>y, \ y+z>x, \ x+y>z$

пред. Правка 2   0
2023-12-06 15:28:09.0 #

Мысли: Давайте подумаем, что вообще определяет тройку чисел из которых можно сделать треугольник? Казалось бы, в треугольнике есть замечательное свойство которое гласит что его стороны $a,b,c$ всегда можно разбить на $(x+y), (y+z), (z+x)$, достигается это разбиение если вписать окружность треугольника и использовать свойство касательной. Но, это вряд ли нам поможет сейчас, перед решением задачи лучше бы отобрать определение которое сыграет нам под руку.

Далее смотрите здесь.

Нарисуем две линии из точки, скажем мы выбрали их длины как $a$ и $b$ ($b>a$), и пусть концы этих двух линий будут соединены линией $c$. Заметим, что если "растягивать" угол между двумя линиями, то и $c$ увеличивается. Если растянуть угол до $180$ градусов, то получится что $c=a+b$. А по теореме косинусов, можно понять что в этом случае $c$ лежит от $0$ до $a+b$ в зависимости от угла. То есть, можно заявить что имея длины $a,b$, что бы третья длина $c$ подходила как сторона, она должна лежать на диапазоне от $0$ до $a+b$ (но не может являться этими значениями).

Решение: Будем отталкиваться от нашего определения, докажем от обратного. Допустим какие-то $t_1,t_2,t_3$ не оказались такой тройкой, и пусть скажем $t_3 \geq t_1+t_2$. Для удобства сделаем замену $A=t_4+t_5+...+t_n$ и $B=\frac{1}{t_4}+\frac{1}{t_5}+...\frac{1}{t_n}.$

Тогда надо будет доказать следующее: $$(t_1+t_2+t_3)(\frac{1}{t_1}+\frac{1}{t_2}+\frac{1}{t_3}) + A\cdot (\frac{1}{t_1}+\frac{1}{t_2}+\frac{1}{t_3})+ B\cdot (t_1+t_2+t_3) +AB \geq n^2+1.$$

пред. Правка 2   0
2023-12-06 08:15:41.0 #

При $n=3$ нам бы пришлось доказать что $(t_1+t_2+t_3)(\frac{1}{t_1}+\frac{1}{t_2}+\frac{1}{t_3})\geq 10$, так что докажем это и используем здесь.

Доказательство факта:

$$(!) \ 3+ \frac{t_1}{t_2}+\frac{t_2}{t_1}+\frac{t_1}{t_3}+\frac{t_3}{t_1}+\frac{t_2}{t_3}+\frac{t_3}{t_2} \geq 10.$$

Можем заметить что очень симметрично и под руку сыграет неравенство $t_3 \geq t_1+t_2$ тогда, когда $t_3=2x, t_1=t_2=x$, будем строить неравенства под это равенство. Тогда сразу избавимся от $\frac{t_1}{t_2}+\frac{t_2}{t_1}$ засунув их в $AM \geq \ GM$: $$\frac{t_1}{t_2}+\frac{t_2}{t_1} \geq 2.$$

$$(!) \ \frac{t_1}{t_3}+\frac{t_3}{t_1}+\frac{t_2}{t_3}+\frac{t_3}{t_2} \geq 5.$$

Понимаю, очень хочется засунуть $ \frac{t_1}{t_3}+\frac{t_3}{t_1}$ и $\ \frac{t_2}{t_3}+\frac{t_3}{t_2}$ под это же неравенство, но мы получим что они будут больше или равны $2+2=4$ что не является $5$, придётся хитрить. Вспомним что то равенство поможет нам подстроить $AM \geq \ GM$, то есть из этих дробей получаем соответственно числа $0.5, 2, 0.5, 2$, значит дроби $\frac{t_3}{t_1}$ и $\frac{t_3}{t_2}$ придётся разделить на $4$ что бы использовать неравенство. $$\Rightarrow \frac{t_1}{t_3}+\frac{t_3}{4t_1} \geq 1, \frac{t_2}{t_3}+\frac{t_3}{4t_2} \geq 1.$$

$$(!) \ \frac{3}{4}(\frac{t_3}{t_1}+\frac{t_3}{t_1}) \geq 3,$$ $$ \Rightarrow (!) \ \frac{t_3}{t_1}+\frac{t_3}{t_1} \geq 4,$$ $$\Rightarrow (!) \ t_3\cdot(t_1+t_2)\geq 4t_1t_2.$$

Наконец-то используем $t_3 \geq t_1+t_2$, $$ t_3\cdot(t_1+t_2)\geq (t_1+t_2)^2 \geq 4t_1t_2 \ (AM \geq \ GM).$$

Вернёмся к задаче: Можно сразу понять что $AB \geq (n-3)^2$ по неравенству КБШ. По сути, это всё что мы имеем про $A$ и $B$, то есть хотелось бы всегда под рукой иметь именно их произведение, в случае с $ A\cdot (\frac{1}{t_1}+\frac{1}{t_2}+\frac{1}{t_3})+ B\cdot (t_1+t_2+t_3)$ хотелось бы к нему прибегнуть, а это ведь можно достигнуть опять же с помощью нашего любимого неравенства о средних: $$ A\cdot (\frac{1}{t_1}+\frac{1}{t_2}+\frac{1}{t_3})+ B\cdot (t_1+t_2+t_3) \geq 2\sqrt{AB} \cdot \sqrt{(t_1+t_2+t_3)(\frac{1}{t_1}+\frac{1}{t_2}+\frac{1}{t_3})} \geq 2\sqrt{10} \cdot (n-3).$$

Давайте применим всё что мы находили ранее: $$(t_1+t_2+t_3)(\frac{1}{t_1}+\frac{1}{t_2}+\frac{1}{t_3}) + A\cdot (\frac{1}{t_1}+\frac{1}{t_2}+\frac{1}{t_3})+ B\cdot (t_1+t_2+t_3) +AB \geq 10+2\sqrt{10} \cdot (n-3)+(n-3)^2,$$

$$\Rightarrow (!) \ 10+2\sqrt{10} \cdot (n-3)+(n-3)^2 \geq n^2+1,$$

$$\Rightarrow (!) \ (2\sqrt{10}-6)(n-3) \geq 0.$$

А это верно при $n \geq 3$.